A conjugate of two permutations question

Click For Summary
The discussion revolves around finding a conjugate permutation a such that x1 = ax2a^-1, with x1 and x2 defined by specific cycles. The solution provided is a = (1 6 8)(2 3 7 5), but the method to arrive at this solution is unclear to the participants. They express confusion about aligning cycles and how to determine the mappings of elements in the permutations. The conversation highlights the challenge of understanding the relationship between the cycles of x1 and x2 and the implications for the conjugate a. Clarification on the reasoning behind the mappings in the permutations is sought to resolve the confusion.
Ryker
Messages
1,080
Reaction score
2

Homework Statement


Suppose x_{1} = \begin{pmatrix}<br /> 2 &amp; 9 &amp; 6 \\<br /> \end{pmatrix}\begin{pmatrix}<br /> 3 &amp; 5 &amp; 8 \\<br /> \end{pmatrix}\begin{pmatrix}<br /> 4 &amp; 7 \\<br /> \end{pmatrix} and x_{2} = \begin{pmatrix}<br /> 1 &amp; 5 &amp; 9 \\<br /> \end{pmatrix}\begin{pmatrix}<br /> 2 &amp; 7 &amp; 6 \\<br /> \end{pmatrix}\begin{pmatrix}<br /> 3 &amp; 4 \\<br /> \end{pmatrix}.

Determine the conjugate a, so that x1 = ax2a-1.

The Attempt at a Solution


I know the solution is a = (1 6 8)(2 3 7 5), since we did this in class. However, we didn't really explain how we got to this solution. And I can do conjugates where you just line up the cycles one under the other, but this method doesn't work here, because, say 1 does not get sent to 2, and 5 not to 9, as you'd assume if you just wrote (1 5 9) above (2 9 6).

I really want to figure this out, but this example really puzzles me, as I haven't yet found the general method, and the fact that x2,i = a(x1,i) doesn't really help me here.

Anyways, any help here would be greatly appreciated.
 
Last edited:
Physics news on Phys.org
I'm not sure what you mean by "line up the cycles" but I think you meant writing x_1 as
\begin{pmatrix}1 &amp; 2 &amp; 3 &amp; 4 &amp; 5 &amp; 6 &amp; 7 &amp; 8 &amp; 9 \\1 &amp; 6 &amp; 5 &amp; 7 &amp; 3 &amp; 9 &amp; 4 &amp; 5 &amp; 2\end{pmatrix}
and x_2 as
\begin{pmatrix}1 &amp; 2 &amp; 3 &amp; 4 &amp; 5 &amp; 6 &amp; 7 &amp; 8 &amp; 9 \\9 &amp; 6 &amp; 4 &amp; 3 &amp; 1 &amp; 7 &amp; 2 &amp; 8 &amp; 5\end{pmatrix}

Do you see how I got that? (I am assuming you are using the convention that your permutations work from right to left.)
 
Last edited by a moderator:
Yeah, I can follow that, but I can't seem to be able to make the next step then. For example, it isn't obvious to me why a sends 1 to 6 instead of say, to 2 or 9.
 
Question: A clock's minute hand has length 4 and its hour hand has length 3. What is the distance between the tips at the moment when it is increasing most rapidly?(Putnam Exam Question) Answer: Making assumption that both the hands moves at constant angular velocities, the answer is ## \sqrt{7} .## But don't you think this assumption is somewhat doubtful and wrong?

Similar threads

Replies
7
Views
2K
  • · Replies 2 ·
Replies
2
Views
1K
  • · Replies 6 ·
Replies
6
Views
1K
  • · Replies 3 ·
Replies
3
Views
2K
  • · Replies 7 ·
Replies
7
Views
2K
  • · Replies 12 ·
Replies
12
Views
3K
  • · Replies 2 ·
Replies
2
Views
2K
  • · Replies 2 ·
Replies
2
Views
977
  • · Replies 2 ·
Replies
2
Views
1K
  • · Replies 2 ·
Replies
2
Views
2K